0
$\begingroup$

Is it possible to use the sieve method to solve problems like these?

  1. Count a number of $p_1p_2\cdots p_r$, a product of $r\geqslant 1$ primes such that $p_i\in [P,2P]$ for all $i=1,2,\ldots, r$ in the short range such as $[AP^r,AP^r+Y]$ for any suitable constant $A\in [1,2^r)$ and $Y=P^{1.9}$ for instance. Heuristically, since the probability of getting a prime in $[P,2P]$ is roughly $\frac{1}{\log P}$, it is quite reasonable to expect the count as $\ll \frac{P^{1.9}}{\log^r P}$?

For $r=2$, the bound is possible by the Prime number theorem in short intervals $$\sum_{\substack{AP^2<pq\leqslant AP^2+Y}\\ p,q\in [P,2P]}1=\sum_{\substack{p\leqslant AP^2+Y\\p\in[P,2P]}}\sum_{\substack{\frac{AP^2}{p}<q\leqslant \frac{AP^2+Y}{p}\\q\in [P,2P]}}1\ll \sum_{\substack{p\leqslant AP^2+Y\\p\in[P,2P]}}\frac{Y}{p\log P}\ll \frac{Y}{P\log P}\sum_{p\in [P,2P]}1\ll \frac{P^{1.9}}{\log^2P}$$ but for higher $r$ the size of $Y$ is too small to adapt.

  1. Given the setting as in 1., Can we show any result on counting a number of pairs of products of $r$ primes such that, for a fixed even number $g=O(P)$, their difference is $g$? Similarly as above, I expect the count to be $\ll\frac{P^{1.9}}{(\log P)^{2r}}$.

What can we say about the implicit constant? Can it be bounded by $\ll (\log P)^{o(1)}$?

$\endgroup$
1
  • $\begingroup$ I think for large $r$ even counting $n_1,\dots,n_r$ such that $n_i \in [P,2P]$ and $n_1 n_2\dots n_r \in [A P^r, A P^r +Y]$ is not so easy. $\endgroup$ Commented Nov 12 at 14:52

0

You must log in to answer this question.

Start asking to get answers

Find the answer to your question by asking.

Ask question

Explore related questions

See similar questions with these tags.